Encontrar los operadores de creación/aniquilación

Usando la firma de Minkowski ( + , , , ) , para la densidad lagrangiana

L = m ϕ m ϕ metro 2 ϕ ϕ

del campo escalar complejo, tenemos el campo

ϕ ( X ) = d 3 pag 2 ( 2 π ) 3 ω pag ( a ( pag ) mi i pag X + b ( pag ) mi + i pag X ) .

Ahora estoy tratando de encontrar una ecuación para el a ( pag ) y b ( pag ) (con el objetivo final de encontrar una expresión para [ a ( pag ) , b ( q ) ] usando [ ϕ ( X ) , Π ( y ) ] ).

Lo que debo señalar es que estamos considerando todo esto en la imagen de Schrödinger (t=0), así que supongo que lo primero que hay que hacer es cambiar todo el X es para X ¿bien?

La estrategia que estoy luchando por implementar y fallando en muchos puntos a lo largo del camino:

  1. Encuentra el impulso Π ϕ ( X ) = L ϕ ˙ = ϕ ˙ .

  2. Añadir alguna combinación de ϕ ( X ) y Π ( X ) para deshacerse de uno de los operadores de creación/aniquilación.

  3. Haz una transformada inversa de Fourier para encontrar a ( pag ) en términos de ϕ ( X ) , Por ejemplo.

Ninguno de los principales libros de texto parece llevar esto a cabo, y en su lugar escriben algo como "y es fácil de mostrar...". Sin embargo, no lo encuentro demasiado fácil, especialmente la parte 3, ya que no soy un experto en transformadas de Fourier.

¿Alguien podría dirigirme a algún lugar donde lo anterior se calcule explícitamente (en más de 2/3 líneas) o ayudarme a comprender cada uno de los 3 pasos anteriores?

(Me doy cuenta de que es fácil encontrar una referencia donde se hace esto para el campo escalar real, en cuyo caso tenemos a ( pag ) y a ( pag ) . Aún así, me resulta difícil seguir las partes).

Entonces, esta me parece una pregunta muy amplia, podría ser útil dividirla en partes más pequeñas. Por ejemplo, ¿qué tienes para la parte 1 (cálculo del impulso)? Debes encontrar que la cantidad de movimiento se conjuga con ϕ es ϕ ˙ y que el impulso se conjuga a ϕ es ϕ ˙ , es eso lo que encuentras?
Sí, pero puedes obtenerlo fácilmente con solo mirar el Lagrangiano. Quizás lo añada entonces
Bien :) Solo trato de averiguar exactamente dónde estás atascado. Está bien, entonces puedes tomar el impulso, y sabes que [ ϕ , Π ] = i d en intervalos de tiempo iguales. puedes expresar Π como una expansión modal de a , b y sus puñales? Si es así, ¿qué obtienes cuando conectas las expansiones de modo de ϕ y Π en [ ϕ , Π ] = i d ? Una vez más, es útil para dar una respuesta para tener una idea más clara de lo que intentó exactamente y dónde las cosas no funcionan.

Respuestas (1)

Los campos satisfacen la ecuación de onda. Por lo tanto, podemos escribir

ϕ ( X ) = d 3 pag ( 2 π ) 3 1 2 ω pag [ a ( pag ) mi i pag X + b ( pag ) mi i pag X ] ϕ ( X ) = d 3 pag ( 2 π ) 3 1 2 ω pag [ b ( pag ) mi i pag X + a ( pag ) mi i pag X ]
dónde ω pag = pag 2 + metro 2 . Invirtiendo esto, encontramos (COMPRUEBE ESTO)
a ( pag ) = i d 3 X mi i pag X 0 ϕ ( X ) b ( pag ) = i d 3 X mi i pag X 0 ϕ ( X ) a ( pag ) = i d 3 X mi i pag X 0 ϕ ( X ) b ( pag ) = i d 3 X mi i pag X 0 ϕ ( X )
dónde A 0 B = A 0 B ( 0 A ) B .

Los momentos conjugados se pueden determinar a partir del Lagrangiano como

π = 0 ϕ ,     π = 0 ϕ
Las relaciones de conmutación en términos de los campos son
[ ϕ ( t , X ) , π ( t , y ) ] = i d 3 ( X y )
Con esta información, debería poder calcular los paréntesis de los coeficientes de moda.

PD: debo agregar que estoy usando el ( + + + ) firma para la métrica.

Buena respuesta; lo más probable es que el OP también necesite la identidad conocida:
d norte ( k ) = d norte X ( 2 π ) norte mi i k X
La manera conceptualmente fácil pero 'algebraicamente difícil' en que lo hice en mi clase de QFT fue simplemente conectar la expansión de Fourier del campo en la relación de conmutación. El uso de todas las relaciones de conmutación de campo/momento le dará la relación correcta entre todos los operadores de creación/aniquilación, sin necesidad de invertir.
@Danu Esto es lo que originalmente comencé a hacer. Una página y media después me di cuenta de que solo iba a obtener algunas relaciones de conmutación de los operadores de creación y aniquilación iguales a i d 3 ( X y ) ?
@Prahar Buena respuesta. Sin embargo, donde dices "verificar esto", este era el principal problema que estaba teniendo. ¿De dónde has sacado esas cuatro ecuaciones? ¿Utilizó el método que sugerí como número 2) en mi pregunta? ¿Existe alguna fórmula general para invertir una transformada de Fourier?
@ user13223423 Sé que este era el principal problema que tenías y te di una fórmula explícita. Todo lo que tiene que hacer es conectar las diversas expansiones y derivar esto. Esencialmente usé el paso 3. De manera más general, usé una técnica más generalizada para definir los operadores de creación-aniquilación en términos de algún producto interno en el espacio de campo clásico.
Se comienza por considerar el espacio clásico de soluciones de la ecuación de onda y definir un producto interno en este espacio ( ϕ 1 , ϕ 2 ) = d 3 X ϕ 1 0 ϕ 2 . Se puede demostrar que este es un producto interno conservado. Luego se elige una base ortonormal en este espacio ϕ k satisfactorio ( ϕ k ϕ k ) = d k , k . Esta base luego se divide en frecuencia positiva. y modos de frecuencia negativa ϕ k ± . Esta distinción (y la base) no es única en general y depende de las coordenadas y del espacio-tiempo.
Por ejemplo, en el caso del espacio de Minkowski, es estándar elegir ϕ k ± = mi i k X . Los operadores de creación y aniquilación se definen entonces como a k = ( ϕ k + , ϕ ) y a k = ( ϕ k , ϕ ) . Trabajando todo esto explícitamente, se obtiene la fórmula que he descrito.
@ user13223423 eso solo significa que cometió un error en alguna parte. Los factores de i debería cancelar (porque π = 0 ϕ )
@Danu, resolveré esto en un momento y, con suerte, llegaré a la respuesta correcta. Pero me preocupa que mi resultado final sea una ecuación con dos incógnitas: a y b . ¿No?
@ user13223423 necesita las cuatro relaciones de conmutación entre los momentos y los campos.
Al usar este enfoque, cuando calculo la relación de conmutación, obtengo el siguiente resultado: [ a ( q ) , a ( pag ) ] = 2 ω pag d 3 ( pag q ) . Sin embargo, cuando me refiero a las notas de David Tong sobre QFT, el CCR para los operadores de creación/aniquilación en la ecuación 2.20 no tiene ω factor. ¿Alguien podría explicarme si estoy cometiendo un error o cuál es el origen de esta confusión?
@newtothis - el factor de 2 ω pag es una cuestión de convención. A algunos autores (como Weinberg y Tong) les gusta hacer la expansión del campo con 1 2 ω pag en lugar de lo que tengo. Me gusta esta convención (usada por Srednicki) porque entonces a ( pag ) se transforma muy bien bajo las transformaciones de Lorentz, mientras que en las convenciones de Weinberg hay raíces cuadradas en todas las fórmulas. Sin embargo, el mayor problema con su fórmula es el signo. Debería ser 2 ω pag . El signo es físicamente importante, el factor general es una convención.
@Prahar está bien, revisaré mi cálculo nuevamente. Si tuviera que definir el campo de la forma en que lo ha hecho, pero con 2 ω pag reemplazando 2 ω pag , no obtendría el factor extra en el resultado final, ¿verdad?
¡Sí! puede ver esto fácilmente simplemente cambiando la escala a ( pag ) 2 ω pag a ( pag ) . Tenga en cuenta que también le falta un factor de ( 2 π ) 3 en algún lugar (¡y el letrero mencionado anteriormente que es mucho más importante!)